A box of crayons costs $1.75, including tax. Mr. Valentino wants to purchase boxes of crayons for his class and has a $25 budget. Write an inequality to solve for the number of boxes of crayons Mr. Valentino can purchase.
NEED HELP PLZZZ :( WILL GIVE BRAINIEST IF RIGHT
A $1.75x ≤ $25
B $1.75x ≥ $25
C $25x ≤ $1.75
D $25x ≥ $1.75

Answers

Answer 1

Answer:

A ) $1.75 x  ≤ $25

Step-by-step explanation:

the cost of x boxes of crayons = 1.75x dollars

If there is a budget of only $ 25,

So, the cost of x boxes of crayons ≤ $25

Answer 2

Answer:

Since his budget is 25, it's less than or equal to ( ≤ 25). That means your answer would be A  $1.75x ≤ $25 because 1.75 times the amount of crayons can't go over the $25 budget.


Related Questions

AT&T charges $350 to get a plan with them, plus $60 per month. T-Mobile charges a $275 initial fee, plus $80 per month. Write an equation or inequality that can be used to determine the number of months it would take for T-Mobile to cost the same or more than AT&T.

Answers

Given :

AT&T charges $350 to get a plan with them, plus $60 per month.

T-Mobile charges a $275 initial fee, plus $80 per month.

To Find :

Write an equation or inequality that can be used to determine the number of months it would take for T-Mobile to cost the same or more than AT&T.

Solution :

Money-time relation of plan AT&T :

[tex]M_1 = 60x + 350[/tex]

Money-time relation of plan T-Mobile :

[tex]M_2 = 80y + 275[/tex]

( Here, x and y are number of months )

Let, after t months would take for T-Mobile to cost the same or more than AT&T.

So, [tex]M_2\ge M_1[/tex]

[tex]80t+275 \ge 60t + 350\\\\20t \ge 350 - 275\\\\20t \ge 75\\\\t \ge 3.75[/tex]

Hence, this is the required solution.

2. Determine the amount of interest owed after 2 years on a $850 loan at 12.25% if no payments
are made. LTD
Enter your answer

Answers

Answer:

Question example:

The amount of interest is 208.74$

Here we have given

Amount (P)=850$

Interest (r)=12.25%

Time (t)=2 years

we have to calculate the simple interest

What is the formula for simple interest?

[tex]I=\frac{PRT}{100}[/tex]

Plug the given value in above formula

[tex]I=\frac{850*12.25*2}{100}[/tex]

Simplify above equation we get

[tex]I=\frac{850*12.25*2}{100}\\\\I=\frac{20874}{100}\\\\I=208.74[/tex]

Therefore the amount of interest is 208.74$

To learn more about the interest visit:

https://brainly.com/question/25793394

Which is equal to 0.47?
CLEAR
SUBMIT
47
1,000
47
100
47
10
47

Answers

Answer: 47/100

Step-by-step explanation: 47/100 = 0.47.






Please help!!!!

Carol's class has ninestudents on roll. newstudents join her class and the number of
students increases to 16. Which equation
represents Carol's class roster.
o 16 + X = -9
oX - 16 = 9
o 16X = 9
09 + X = 16

Answers

The last one, 9+x=16

Answer:

9 + X = 16

Step-by-step explanation:

Given parameters:

   Number of students on the roll  = 9

   

Unknown:

The equation representing the class roster  = ?

Solution:

Let the number of additional students that joined the class = x

  Carol's class roster:

     New number of students = number of students + additional students

          16  = 9 + x

The solution of the problem is 9 + x = 16

This recipe makes 15 cupcakes.
Deepak only wants to make 12 cupcakes.
How much of each ingredient does he need?

Recipe: Makes 15 cupcakes
12 ounces butter
10 ounces sugar
15 ounces flour
5 eggs
I WILL GIVE BRAINLIEST TO THE BEST ANSWER IF ITS RIGHT

Answers

He will only need 3 ounces of butter, 1 ounce of sugar, 3 ounces of flour, and 5 eggs hope I helped


Convert y = x2 - 4x + 1 to vertex form by completing the square.

O y= (- 1)2 + 4
O y= (1 - 2)2 +5
O y= (x - 2)2 – 3
O y = (x - 4) + 1

Answers

Answer:

vertex: (-2, -3)

Step-by-step explanation:

y = x2 + 4x                 + 1  

y = x2 + 4x + 22          + 1 - 22  

y = (x + 2)2 + 1 - 4  

y = (x + 2)2 - 3            in y = (x - h)2 + k, (h, k) is the vertex.  

 

Answer:

y = (x - 2)² - 3

Step-by-step explanation:

Given

y = x² - 4x + 1

To complete the square

add/subtract ( half the coefficient of the x- term )² to x² - 4x

y = x² + 2(- 2)x + 4 - 4 + 1

   = (x - 2)² - 3

Write the linear equation in slope intercept form y-2=1/2(x+4)

Answers

Answer:

y = 1/2x + 4

Step-by-step explanation:

Slope-Intercept Form: y = mx + b

m - slope

b - y-intercept

Step 1: Define

Point-slope form; y - 2 = 1/2(x + 4)

Step 2: Rewrite

Distribute 1/2 to both sides: y - 2 = 1/2x + 2Add 2 to both sides: y = 1/2x + 4

Answer:

y=1/2x+4

Step-by-step explanation:

y-2=1/2(x+4)

(distribute the 1/2)

y-2=1/2x+2

(add 2 to both sides)

y=1/2x+4

Hummingbirds beat their wings very fast. The smaller the
hummingbird is, the faster its wings beat. The average hummingbird beats its
wings about 3 x 10 3 times a minute. How many times a minute is that, written as
a whole number?

Pls help me! I will give brainliest

Answers

Answer:

A hummingbird's heart beats from 225 times per minute when it's at rest and more than 1,200 times per minute when it is flying. Its wings beat about 70 times per second in regular flight and more than 200 times per second while diving.

Answer: A hummingbird's heart beats from 225 times per minute when it's at rest and more than 1,200 times per minute when it is flying. Its wings beat about 70 times per second in regular flight and more than 200 times per second while diving.

Step-by-step explanation:

a population of bears is decreasing the population this year is 200 bears after 1 year it is estimated that the population will be 160 bearsafter 2 years it is estimated that the population will be 128 bears what equation describes the bear population in any year x?

Answers

Answer:

200x-160

Step-by-step explanation:

Substitute the given value for x into each equation to determine the value of y.
1. y = 3x + 6; x = -4
2.3.2 - 8y = 12; x = 4
3. y = -43; x = -2

Answers

Answer:

Y=-6

Y= -1.1

Y=-43

Step-by-step explanation:

Answer for brainliest.

Answers

Answer:

t > 28

Step-by-step explanation:

Given

[tex]\frac{t}{4}[/tex] > 7 ( multiply both sides by 4 to clear the fraction )

t > 28

Answer:

[tex]t>28[/tex]

Step-by-step explanation:

[tex]\frac{t}{4} >7\\[/tex]

Times both sides by 4

[tex]t>28[/tex]

Hope this helps and pls do mark me brainliest:)

bailey ran 0.674 times aound her school track. what percentage of the track did she run around?

f: 0.0674%
G: 6.74%
H:64.4%
j: 674%

Answers

Answer:

0.674 as a percent. 0.674 is equivalent to 67.4%.

Hope this helps!

Calculate the reciprocal of -0.013, correct to 1 decimal

Answers

Answer:

Step-by-step explanation:

-0.013 = [tex]\frac{-13}{1000}[/tex]

Reciprocal = [tex]\frac{-1000}{13} =-76.9[/tex]

Which equation represents a line that passes through (5, 1) and has a slope of 1/2?

y-5= 1/2(x-1)

y-1/2=5(x-1)

y-1=1/2(x-5)

y-1=5[x-1/2]​

Answers

Step-by-step explanation:

Hey there!

The given point is (5,1) and slope is 1/2.

Let's use one point formula for finding the eqaution.

[tex](y - y1) = m(x - x1)[/tex]

Keep all values.

[tex](y - 1) = \frac{1}{2} (x - 5)[/tex]

This matches with Option C,

So, your answer is Option C.

Hope it helps...

Which of the following situations have a sum of zero? Select all that apply.

1. Shawna spends $20 on materials to make bracelets for her friends. She then earns $20 from selling the bracelets.

2. A diver dove down 20 1/4 feet under water and then descended 20 1/4 feet.

3. A golfer shot 2 over par (+2) on a hole. They then then shot 2 under par (−2) on the next hole.

4. You have $5.50 in your pocket and then you receive $5.50 for doing your chores.

Answers

I’m pretty sure it’s 1. And 2.
The first and the third.

plz help i dont know the answer

Answers

Answer:

Im not 100 percent sure but I think you have to do this

Step-by-step explanation:

Add all numbers together to find total students

Then add the students in Chemistry and Physics and both together

Then find what that number is in percent to the total students

A and \angle B∠B are supplementary angles. If m\angle A=(5x-19)^{\circ}∠A=(5x−19)

and m\angle B=(2x+10)^{\circ}∠B=(2x+10)

, then find the measure of \angle B∠B.

Answers

Answer: 52

Step-by-step explanation:

4x+8 = 5x-3 by the Vertical Angles Theorem that states the vertical angles are congruent.  

Subtract 4x from both sides and get:

8=1x -3

Add 3 to both sides:

11= 1x

if x =11, plug that in to angle A.  

4x+ 8 becomes 4(11)+8 =

44+ 8=

52

There are (64)2 ⋅ 60 pigs on a farm. What is the total number of pigs on the farm?

Answers

I’m bot sure if you ask 64^2 x 60 or 64 x 2 x 60. If you ask the first one, the answer is 245760. But if you ask the second one, the answer is 7680.

Answer:

If the question is (64)2 x 60, then the answer is 7,200

Step-by-step explanation:

To encourage recycling many states require a $.70 deposit or drink containers the total deposit di you pay depends on the number of containers N You by the function D =0.07N could be used to describe the situation if you deposit $1.40 how many containers do you buy?

Answers

Answer:

20 containers

Step-by-step explanation:

Given:

D = 0.07N

Where,

D = Amount of deposits

N = Number of containers

If D = $1.40 find N

D = 0.07N

1.40 = 0.07N

Divide both sides by 0.07

N = 1.40 / 0.07

= 20

N = 20 containers

if you deposit $1.40, you will buy a total of 20 containers.

8c + 20 = -180

c =

can some one help me please

Answers

Answer:

c = -25

Step-by-step explanation:

8c + 120 = -180

Subtract 20 from both sides.

8c = -200

Divide both sides by 8.

c = -25

Answer:

C = -25

Step-by-step explanation:

Start by getting C alone, subtract 20 from the left side of the equal sign and subtract 20 from the right. (what you do on one side you MUST do to the other) this gives you 8c= -200. Divide 8 from 8c on the right side and divide 8 from -200 on the left side. This will give you c= -25

determine the unit rate for each ratio.
The gas station charges $180 for 3 hours of work. How much
they charge for 1 hour of work?

Answers

Answer:

$60

Step-by-step explanation:

180/3 = 60

I hope this helps :)

Answer:60 per hour

Step-by-step explanation: you divide 180 by 3

PLEASE HELP WILL MARK BRAINLIEST FOR RIGHT ANSWER!!

Answers

Answer:

Population 2 pattern is that every year the population goes down by 2

Step-by-stepation:

What is the value for y
When x=20
Y=-2x+8

Answers

Answer:

y = -32

Step-by-step explanation:

Y=-2x+8

Let x= 20

y = -2 * 20 +8

y = -40+8

   = -32

8.
Which of the following is a negative
rational number that is not an integer?
А -pi
B -4.5
C -2
D 5 squared

Answers

The answer is B, 4.5
It is not an integer because it’s a decimal, it’s a rational number, unlike pi, and it’s negative, so it the answer is 4.5

Exercise 1. A truck has the capacity to transport up to 8 tons of a material at a time. If 104 tons of the material should be transported, how many trucks like this will be needed?

Answers

Answer:

how many trucks ?

Step-by-step explanation:

It is solved using the rule of three simple ones, where the number of trucks is for the quantity of material transported. We have that 1 is for 8 as X is for 104 (Look attached).

Multiplying cross, we have to:

8X = 104

X = 104/8

X= 13

If the 16 boys in the classroom are 40% of the class how many students are there in the class

Answers

Answer:

56

Step-by-step explanation:

40+16=56

Answer:

40

Step-by-step explanation:

Set up the equation.

Let the full number of people in the classroom = x

(40%) x = 16                Change 40% to a fraction

(40/100) x = 16            Reduce 40/100 to a decimal

0.4 x = 16                     Divide by 0.4

0.4/0.4 x = 16/0.4

x = 40

Al saves pennies. He agreed to give 3/7 of his pennies to Bev if she would give 3/7 of what she got from Al to Carl and if Carl in turn would give 3/7 of what he got from Bev to Dani.​ Bev, Carl, and Dani agreed and Dani received pennies. How many pennies did Al have​ initially

Answers

Answer:

2744

Step-by-step explanation:

Let Al had x pennies initially.

Al agreed to give 3/7 of his pennies to Bev if she would give 3/7 of what she got from Al to Carl and if Carl in turn would give 3/7 of what he got from Bev to Dani.

So, as per the given condition,

The number of Pennies received by Ben from Al [tex]= \frac{3}{7} x[/tex]

The number of Pennies received by Carl from Bev

[tex]=\frac{3}{7} \times \frac{3}{7} x=\frac{9}{49}x[/tex]

The number of Pennies received by Dani from Carl

[tex]= \frac{3}{7}\times \frac{9}{49}x=\frac{27}{343} x[/tex]

Now, as Dani received 216 Pennies, so

[tex]\frac{27}{343} x =216[/tex]

[tex]\Rightarrow \frac{216\times 343}{27}=2744.[/tex]

Hence, initially, Al had 2744 Pennies.

Students was 1: 8. However, this year the ratio of the number of middle school students to the number of high school students changed to 2: 7. If there are 18 middle school students in the band this year, how many fewer high school students are in the band this year compared to last year? Expla

Answers

Answer:

Step-by-step explanation:

Last year

Ratio of the number of middle school students to number of high school students = 1 : 8

Total ratio = 9

Middle school = 1

High school = 8

Total students in the band = x

Middle school = 1/9x

High school = 8/9x

Total = 1/9x + 8/9x

This year

Ratio of the number of middle school students to number of high school students = 2 : 7

Total ratio = 9

Middle school = 2

High school = 7

Total students = x

Middle school = 2/9x

High school = 7/9x

Total = 2/9x + 7/9x

If there are 18 middle school students in the band this year

2/9x = 18

x = 18÷2/9

= 18 × 9/2

= 81

x = 81

Last year:

1/9x + 8/9x

= 1/9 * 81 + 8/9 * 81

= 9 + 72

= 81

This year

2/9x + 7/9x

2/9*81 + 7/9*81

= 18 + 63

=81

Change in middle school =this year - last year

= 18 - 9

= 9 (increase)

Change in high school = this year - last year

= 63 - 72

= -9 (decrease)

14.) George is designing a pool that will be 34 feet long. He plans to make
a scale drawing of the pool by letting 1/4 inch represent 1 foot. How long
should the line representing the length of the pool in the diagram be?

Answers

Answer:

I believe the answer is 8.5 inches long

Step-by-step explanation:

we have to find out how long 1/4 × 34 is, since each 1/4 represents one of the 34 feet of the pools length.

34×.25 = 8.5

3/4 change 3/4 to a decimal show work

Answers

Answer:

0.75

Step-by-step explanation:

You just need to convert 3/4 into a decimal.

For example: If you get 3 out of 4 problems right on a quiz, in percent form it will be a 75%, in decimal form 0.75 because you'll move the percent sign 2 spaces to the left. That will then be a decimal if you're converting 75% to 0.75.

3 divided by 4 is also going to give you 0.75. Because, a "fraction slash" is the dividing point. So,

3/4 = 0.75

Other Questions
An angle measures 50 less than the measure of its supplementary angle. What is the measure of each angle? what is the value of y is 77.65488319 rational or irrational The points A, B, C, and D are plotted on a line, consecutively. AB = BC = CD = 6 cm. Find the distance between M and N, which are the midpoints of segments AB and CD , respectively. Fill in the blanks of the Statement/Reason solution.Statement Reason 1. AM = BM = AB 2 =_____ cm 1. ____________________ 2. CN = _____ = CD 2 = _____ cm 2. Def. of midpoint3. MN = MB + ______ + CN 3. ____________________4. MN = ________ cm 4. Algebra The answer has to be at least 4 sentences long. I am literally crying at this time please help me guys please I will mark brainliest please guys help I honestly beg you guys. Who serves as the commander in chief of the United States military forces What is the equation in slope-intercept form of the line that passes through the points (12, 4) and (4, 8)?A. y = 43 20A. , y = 43 20B. y = 43 12B. , y = 43 12C. y = 0.75 13C. , y = 0.75 13D. y = 0.75x +5D. , y = 0.75x +5 what is the rate of change of y=7x+9 sub to astronaut gaming i have 27 or 28 subs if you need help finding i will help Janet and Jack are walking in a two-day event to raise money for charity. Janet asks a friend for a $7.50 donation, plus $0.10 for each mile she walks. Jack asks a friend for a $5.50 donation, plus $0.15 for each mile he walks. At how many miles of walking would Janet and Jack collect the same amount of money? a Clydesdale drinks about 120 gallons of water every 4 days. At this rate how many gallons of water does a Clydesdale drink in 28 days? A transform boundary is marked by a strike-slip fault like the ______ fault. Isabella is having a party with 45 balloons. She needs 6 feet of string to attach toeach balloon. How much string would she need if it is sold by the yard? (Hint: 1yard = 3 feet.) Number Sense Write an additionsentence and a multiplicationsentence to solve this problem.Jessie bought 4 packages of colorfulstones to put in the fish bowl. Thereare 6 stones in each package. Howmany stones did Jessie buy? The members of the gardening group plan to build a walkway through the garden as formed by the hypotenuse of each of the four triangles in the drawing. That way, the gardeners will be able to access all sections of the garden. Calculate the length of the entire walkway to the nearest hundredth of a yard. Show your work Can someone please help me with this question (there are more questions like this that I will post later, if your answer is correct I will give you brainliest) ASAPWhat do the different categories of hurricanes represent?O Air densityO HumidityO PressureO Wind speeds Which answer choice correctly describes how the US Constitution fixed a problem or problems of the Articles of Confederation?O A Under the Constitution, the people could directly elect their representatives in the US Senate.OB. Under the Constitution, the state governments were granted more control over immigration and trade.OC. Under the Constitution, the federal government had the power to enforce laws but was checked by Congress and the court systOD. Under the Constitution, political candidates must be a member of a political party so that they represent the interests of thepeople. Mountain ranges are formed from what process? equivalent fractions for 2 1/5 and 1 5/6 with common denominator 30